Business Statistics and Mathematics Solved Paper 2013, Punjab University, BCOM, ADC I

Business Statistics and Mathematics Solved Paper 2013, Punjab University, BCOM, ADC I

In this Post, we are going to discuss the Paper of Business Statistics and Mathematics Solved Paper 2013Punjab University, BCOM, ADCI in which Measures of Central TendencyMeasures of DispersionCorrelation & Regression, Index Numbers, Matrix, Arithmetic Progression, Geometric Progression, Simultaneous Linear Equations, Annuity, Quadratic Equation is discussed and solved.

Other solved papers of Business Statistics & Mathematics

Business Statistics and Mathematics Solved Paper 2013, Punjab University, BCOM, ADC I

Table of Contents

Section I Business Statistics

Q.1 Weight of 175 male students at a university are given in the following frequency table…..Calculate Karl Pearson’s and Bowley’s Coefficient of Skewness.

Q.1 Weight of 175 male students at a university are given in the following frequency table:

WeightFrequency
118-12620
127-13535
136-14449
145-15332
154-16225
163-17114

Calculate Karl Pearson’s and Bowley’s Coefficient of Skewness

Solution:

ClassesClass BoundariesXFrequency (f)fxfx²C.F
118-126117.5-126.512220244029768020
127-135126.5-135.513135458560063555
136-144135.5-144.5140496860960400104
145-153144.5-153.5149324768710432136
154-162153.5-162.5158253950624100161
163-171162.5-171.5167142338390446175
   ∑f = n= 175 ∑f = 24941∑ fx² = 3583693 

    \[ \mathbf{A.Mean\ }\overline{\mathbf{X}}\mathbf{=}\frac{\mathbf{\sum fx}}{\mathbf{\sum f}}\mathbf{=}\frac{\mathbf{24941}}{\mathbf{175}}\mathbf{= 142.52}\  \]

Selection of Model Class for Median

    \[ \frac{\mathbf{n}}{\mathbf{2}}\mathbf{=}\frac{\mathbf{175}}{\mathbf{2}}\mathbf{= 87.5\ falls\ in\ C.F\ 104}\  \]

    \[  \mathbf{Meadian = L +}\frac{\mathbf{h}}{\mathbf{f}}\mathbf{(}\frac{\mathbf{n}}{\mathbf{2}}\mathbf{- \ C)}\ \]

    \[ \mathbf{Meadian = 135.5 +}\frac{\mathbf{9}}{\mathbf{49}}\left( \mathbf{87.5 - \ 55} \right)\  \]

    \[ \mathbf{Meadian = 135.5 +}\frac{\mathbf{9}}{\mathbf{49}}\left( \mathbf{32.5} \right)\  \]

    \[ \mathbf{Meadian = 141.46}\  \]

Model Class for Mode

Maximum Frequency is 49 so L = 135.5, fm =49, f1 =35, f2 =32 & h=9

    \[ \mathbf{Mode = L +}\frac{\mathbf{(fm - f}\mathbf{1)}}{\left( \mathbf{fm - f}\mathbf{1} \right)\mathbf{+ \ (fm - f}\mathbf{2)}}\mathbf{\ \times \ h}\  \]

    \[  \mathbf{Mode = 135.5 +}\frac{\mathbf{(49 - 35)}}{\left( \mathbf{49 - 35} \right)\mathbf{+ \ (49 - 32)}}\mathbf{\ \times \ 9}\ \]

    \[ \mathbf{Mode = 135.5 +}\frac{\mathbf{14}}{\mathbf{31}}\mathbf{\times 9\ }\  \]

    \[ \mathbf{Mode = 139.56\ })  \]

Selection of Model Class for Q1

    \[  \frac{\mathbf{1}\mathbf{n}}{\mathbf{4}}\mathbf{=}\frac{\mathbf{175}}{\mathbf{4}}\mathbf{= 43.75\ falls\ in\ C.F\ 55}\ \]

    \[  \mathbf{Q}\mathbf{1 = L +}\frac{\mathbf{h}}{\mathbf{f}}\left( \frac{\mathbf{n}}{\mathbf{4}}\mathbf{- \ C} \right)\ \]

    \[  \mathbf{Q}\mathbf{1 = 126.5 +}\frac{\mathbf{9}}{\mathbf{35}}\left( \mathbf{43.75 - \ 20} \right)\ \]

    \[ \mathbf{Q}\mathbf{1 = 126.5 +}\frac{\mathbf{9}}{\mathbf{35}}\left( \mathbf{43.75 - \ 20} \right)\  \]

Selection of Model Class for Q3

    \[  \frac{\mathbf{3}\mathbf{n}}{\mathbf{4}}\mathbf{=}\frac{\mathbf{3(175)}}{\mathbf{4}}\mathbf{= 131.25\ falls\ in\ C.F\ 136}\ \]

    \[ \mathbf{Q}\mathbf{3 = L +}\frac{\mathbf{h}}{\mathbf{f}}\left( \frac{\mathbf{3}\mathbf{n}}{\mathbf{4}}\mathbf{- \ C} \right)\  \]

    \[ \mathbf{Q}\mathbf{3 = 144.5 +}\frac{\mathbf{9}}{\mathbf{32}}\left( \mathbf{131.25 - \ 104} \right)\  \]

    \[  \mathbf{Q}\mathbf{3 = 152.16}\ \]

    \[ \mathbf{S.D =}\sqrt{\frac{\mathbf{\sum fx²}}{\mathbf{\sum f}}\mathbf{-}\left( \frac{\mathbf{\sum fx}}{\mathbf{\sum f}} \right)^{\mathbf{2}}}\  \]

    \[ \mathbf{S.D =}\sqrt{\frac{\mathbf{3583693}}{\mathbf{175}}\mathbf{-}\left( \mathbf{142.52} \right)^{\mathbf{2}}}\  \]

    \[  \mathbf{S.D =}\sqrt{\mathbf{20,478.24 - 20,311.95}\mathbf{\ }}\ \]

    \[ \mathbf{S.D =}\sqrt{\mathbf{166.29}}\mathbf{= 12.89}\  \]

    \[ \mathbf{Karl\ Pearso}\mathbf{n}^{\mathbf{'}}\mathbf{s\ Coefficient\ of\ \ Skewness = \ }\frac{\mathbf{Mean - Mode}}{\mathbf{S.D}}\  \]

    \[ \mathbf{Karl\ Pearso}\mathbf{n}^{\mathbf{'}}\mathbf{s\ Coefficient\ of\ \ Skewness = \ }\frac{\mathbf{142.52 - 139.56}}{\mathbf{12.89}}\  \]

    \[ \mathbf{Karl\ Pearso}\mathbf{n}^{\mathbf{'}}\mathbf{s\ Coefficient\ of\ \ Skewness = \ 0.2296}\  \]

    \[  \mathbf{Bowly's\ Quartile}\mathbf{\ }\mathbf{\ Coefficient\ of\ \ Skewness = \ }\frac{\mathbf{Q}\mathbf{3 + Q}\mathbf{1 - 2}\mathbf{Median}}{\mathbf{Q}\mathbf{3 - Q}\mathbf{1}}\ \]

    \[ \mathbf{Bowly's\ Quartile}\mathbf{\ }\mathbf{\ Coefficient\ of\ \ Skewness = \ }\frac{\mathbf{152.16 + 132.6 - 2(141.46)}}{\mathbf{152.16 - 132.6}}\  \]

    \[  \mathbf{Bowly's\ Quartile}\mathbf{\ }\mathbf{\ Coefficient\ of\ \ Skewness = \ }\frac{\mathbf{284.76 - 282.92}}{\mathbf{19.56}}\ \]

    \[ \mathbf{Bowly's\ Quartile}\mathbf{\ }\mathbf{\ Coefficient\ of\ \ Skewness = \ 0.0940}\  \]

Q.2 Calculate weighted index number of prices for the year 2012 from the following data taking 2008 as base and using formulae recommended by: Laspeyre, Fisher, Paasche’s and Marshall

YearABC
PriceQuantityPriceQuantityPriceQuantity
20085.0803.6903.120
20128.71005.7954.630

Solution:

Commodity20082012 
Price PoQuantity q0Price P1Quantity q1p0q0p1q1P1q0P0q1
A5808.7100400870696500
B3.6905.795324541.5513342
C3.1204.630621389293
Sum7861549.51301935
 ∑p0q0=∑p1q1=∑P1q0=∑P0q1=

    \[ \left( \mathbf{i} \right)\mathbf{Laspeyr}\mathbf{e}^{\mathbf{'}}\mathbf{s\ Index\ 2012}\mathbf{=}\frac{\mathbf{\sum p}\mathbf{1}\mathbf{q}\mathbf{0}}{\mathbf{\sum p}\mathbf{0}\mathbf{q}\mathbf{0}}\mathbf{\times \ 100}\  \]

    \[ \mathbf{Laspeyr}\mathbf{e}^{\mathbf{'}}\mathbf{s\ Index\ 2012}\mathbf{=}\frac{\mathbf{1301}}{\mathbf{786}}\mathbf{\ \times \ 100 = \ 165.52}\  \]

    \[  \left( \mathbf{ii} \right)\mathbf{Paasch}\mathbf{e}^{\mathbf{'}}\mathbf{s\ Index\ 2012}\mathbf{=}\frac{\mathbf{\sum p}\mathbf{1}\mathbf{q}\mathbf{1}}{\mathbf{\sum poq}\mathbf{1}}\mathbf{\times \ 100}\ \]

    \[ \mathbf{Paasch}\mathbf{e}^{\mathbf{'}}\mathbf{s\ Index\ 2012}\mathbf{=}\frac{\mathbf{1549.5}}{\mathbf{935}}\mathbf{\times \ 100 = 165.72}\  \]

    \[  \left( \mathbf{iii} \right)\mathbf{Fishe}\mathbf{r}^{\mathbf{'}}\mathbf{s\ Ideal\ Index\ 2012 =}\sqrt{\mathbf{L \times P}}\ \]

    \[ \mathbf{Fishe}\mathbf{r}^{\mathbf{'}}\mathbf{s\ Ideal\ Index\ 2012 =}\sqrt{\mathbf{165.52 \times 165.72}}\  \]

    \[ \mathbf{Fishe}\mathbf{r}^{\mathbf{'}}\mathbf{s\ Ideal\ Index\ 2012 = 165.61}\  \]

    \[  \left( \mathbf{iv} \right)\mathbf{\ Marsha}\mathbf{l}^{\mathbf{'}}\mathbf{s\ Index\ 2012 =}\left( \frac{\mathbf{\sum p}\mathbf{1}\mathbf{q}\mathbf{0 + \sum p}\mathbf{1}\mathbf{q}\mathbf{1}}{\mathbf{\sum p}\mathbf{0}\mathbf{q}\mathbf{0 + \sum p}\mathbf{0}\mathbf{q}\mathbf{1}} \right)\mathbf{\times 100}\ \]

    \[  \mathbf{Marsha}\mathbf{l}^{\mathbf{'}}\mathbf{s\ Index\ 2012 =}\left( \frac{\mathbf{1301 + 1549.5}}{\mathbf{786 + 935}} \right)\mathbf{\times 100}\ \]

    \[  \mathbf{Marsha}\mathbf{l}^{\mathbf{'}}\mathbf{s\ Index\ 2012 =}\left( \frac{\mathbf{2850.5}}{\mathbf{1721}} \right)\mathbf{\times 100}\ \]

    \[  \mathbf{Marsha}\mathbf{l}^{\mathbf{'}}\mathbf{s\ Index\ 2012 = 165.63}) \]

Q.3. A survey of 1600 families was conducted to observe that high and low income people send children to private and government school. The following results were obtained….Test whether income and type of school are independent at 5% level of significance (table Value is 3.841)

Q.3. A survey of 1600 families was conducted to observe that high and low income people send children to private and government school. The following results were obtained:

IncomeSchoolTotal
PrivateGovernment
High4945061000
Low162438600
Total6569441600

Test whether income and type of school are independent at 5% level of significance (table Value is 3.841)

Solution

(i) Testing the Hypothesis

Ho: There is no Association between Income & School Choice.

H1: There is Association between Income & School Choice.

(ii) Level of Significance = α=0.05

(iii) Test Statistics is:

    \[  \mathbf{\chi^2 = \ \sum}\frac{\mathbf{(fo - fe)}^{\mathbf{2}}}{\mathbf{fe}}\ \]

(iv) To find χ² first calculate expected frequencies fe:

Calculation of expected frequencies (fe)

IncomeSchoolTotal
PrivateGovernment
High

    \[ \frac{1000 \times 656}{1600} = 410\  \]

    \[ \frac{1000 \times 944}{1600} = 590\ \]

1000
Low

    \[ \frac{600 \times 656}{1600} = 246  \]

    \[ \frac{600 \times 944}{1600} = 354\  \]

600
Total6569441600

(v) Calculation of χ²:

Computation of χ²

fofefo-fe(fo-fe)²

    \[  \frac{\mathbf{(fo - fe)}^{\mathbf{2}}}{\mathbf{fe}}\ \]

49441084705617.210
506590-84705611.959
162246-84705628.683
43835484705619.932
     
  77.784
    

    \[  \mathbf{\sum}\frac{\left( \mathbf{fo - fe} \right)^{\mathbf{2}}}{\mathbf{fe}}\mathbf{=}\ \]

(vi) Critical Region: Degree of Freedom d.f= (R-1)(C-1)

So d.f= (2-1)(2-1)=1

The Value of Tabulated χ²(0.05,1)=3.841

The Critical Region χ²cal>3.841

(vii) Conclusion: The calculated value of χ² is 77.784 is greater than the tabulated value of χ² 3.841 or 77.784 falls in the critical region. We reject the Null Hypothesis and accept alternative hypothesis. We can say that there is relationship between Income level and choice of school.

Business Statistics and Mathematics Solved Paper 2013, Punjab University, BCOM, ADC I

Q.4. Given the six elements population 0, 3, 6, 12, 15 and 18. How many samples of size n = 3 can be drawn without replacement from this population. Form sampling distribution of sample means. Hence state and verify the relation between…..

Q.4. Given the six elements population 0, 3, 6, 12, 15 and 18. How many samples of size n = 3 can be drawn without replacement from this population. Form sampling distribution of sample means. Hence state and verify the relation between:

(i) Mean of the sampling distribution of the means and the population mean.

(ii) Variance of the sampling distribution of the mean and population variance.

Solution:

Population = 0,3, 6, 12, 15, 18

Population Size N = 6

Sample size n = 3

    \[ Sample\ Space\ \eta(S) = \begin{pmatrix}N \\C \\n \\\end{pmatrix} = \begin{pmatrix}6 \\C \\3 \\\end{pmatrix} = 20\ \]

Sampling Distribution of Sample Means

S/NoSamplesSum of SamplesMean of SamplesS/NoSamplesSum of SamplesMean of Samples
10,3,693113,6,12217
20,3, 12155123,6,15248
30,3, 15186133,6, 18279
40,3, 18217143,12,153010
50,6, 12186153, 12, 183311
60,6, 15217163, 15, 183612
70,6, 18248176, 12, 153311
80, 12, 15279186, 12, 183612
90, 12, 183010196, 15, 183913
100, 15, 1833112012, 15, 184515

Sampling Distribution of Sample Means

ffX̅fX̅²
3139
51525
621272
7321147
8216128
9218162
10220200
11333363
12224288
13113169
15115225
 201801788
 ∑f =∑fX̅ =∑fX̅²=

Mean & Variance of Sampling Distribution

    \[ µ\overline{x\ } = \frac{\sum f\overline{x\ }}{\sum f} = \frac{180}{20} = 9\  \]

    \[ \sigma{\overline{x\ }}^{2} = \frac{\sum f\overline{x\ }²}{\sum f} - \left( \frac{\sum f\overline{x}}{\sum f} \right)^{2}\  \]

    \[ \sigma{\overline{x\ }}^{2} = \frac{1788}{20} - (9)^{2}\  \]

    \[ \sigma{\overline{x\ }}^{2} = \frac{1788}{20} - (9)^{2}\  \]

    \[  \sigma{\overline{x\ }}^{2} = \frac{1788}{20} - (9)^{2}\ \]

Mean & Variance of Population

X
00
39
636
12144
15225
18324
54738
∑X =∑X² =

    \[ Mean\ µ = \ \frac{\sum X}{N} = \frac{54}{6} = 9\  \]

    \[ \sigma^2 = \frac{\sum X²}{N} - \left( \frac{\sum X}{N} \right)^{2}\  \]

    \[  \sigma^2 = \frac{738}{6} - (9)^{2}\ \]

    \[ \sigma^2 = 123 - 81\  \]

    \[ \sigma^2 = 42\  \]

Verification Formulas:

    \[ µ\overline{x}\ = \ µ\  \]

    \[ 9 = 9\  \]

    \[  {\sigma\overline{x}}^{2} = \frac{\sigma ²}{n}\left\lbrack \frac{N - n}{N - 1} \right\rbrack\ \]

    \[ 8.4 = \frac{42}{3}\left\lbrack \frac{6 - 3}{6 - 1} \right\rbrack\  \]

    \[ 8.4 = 8.4\  \]

Section II Business Mathematics

    \[  Q.5:\;If\;A=\begin{bmatrix}1&3&5\\4&-2&7\\3&2&-4\end{bmatrix}then\;obtain\;A^{-1}\;inverse\;of\;A. \]

Solution:

    \[  A^{- 1} = \frac{Adj.\ A}{|A|}\ \]

Calculation of Determinant

    \[  |A| = \left| \begin{matrix}1 & 3 & 5 \\4 & - 2 & 7 \\3 & 2 & - 4 \\\end{matrix} \right|\ \]

    \[  |A| = 1\left| \begin{matrix}2 & 7 \\2 & - 4 \\\end{matrix} \right| - 4\left| \begin{matrix}3 & 5 \\2 & - 4 \\\end{matrix} \right| + 3\left| \begin{matrix}3 & 5 \\2 & 7 \\\end{matrix} \right|\ \]

    \[  |A| = 1(8 - 14) - 4( - 12 - 10) + 3\lbrack 21 - ( - 10)\rbrack\ \]

    \[ |A| = 1( - 6) - 4( - 22) + 3(31)\ \]

    \[ |A| = - 6 + 88 + 93\ \]

    \[ |A| = 175\ \]

Calculation of Adj. A

Step 1 Calculation of Minors

    \[ \begin{bmatrix}\left| \begin{matrix}2 & 7 \\2 & - 4 \\\end{matrix} \right| & \left| \begin{matrix}4 & 7 \\3 & - 4 \\\end{matrix} \right| & \left| \begin{matrix}4 & - 2 \\3 & 2 \\\end{matrix} \right| \\\left| \begin{matrix}3 & 5 \\2 & - 4 \\\end{matrix} \right| & \left| \begin{matrix}1 & 5 \\3 & - 4 \\\end{matrix} \right| & \left| \begin{matrix}1 & 3 \\3 & 2 \\\end{matrix} \right| \\\left| \begin{matrix}3 & 5 \\2 & 7 \\\end{matrix} \right| & \left| \begin{matrix}1 & 5 \\4 & 7 \\\end{matrix} \right| & \left| \begin{matrix}1 & 3 \\4 & - 2 \\\end{matrix} \right| \\\end{bmatrix} \]

    \[ \begin{bmatrix}6 & - 37 & 14 \\22 & - 19 & - 7 \\31 & - 13 & - 14 \\\end{bmatrix}\  \]

Step 2: Calculation of Cofactors

    \[ \begin{bmatrix}6 & - 37 & 14 \\22 & - 19 & - 7 \\31 & - 13 & - 14 \\\end{bmatrix} \times \begin{bmatrix}+ & - & + \\- & + & - \\+ & - & + \\\end{bmatrix}\ \]

    \[ \begin{bmatrix}6 & 37 & 14 \\22 & - 19 & 7 \\31 & 13 & - 14 \\\end{bmatrix}\ \]

Step 3 Transpose of Cofactors = Adjoint A

    \[ \begin{bmatrix}6 & 22 & 31 \\37 & - 19 & 13 \\14 & 7 & - 14 \\\end{bmatrix}\ \]

    \[ A^{- 1}\mathbf{=}\frac{\mathbf{Adj.\ A}}{\left| \mathbf{A} \right|}\ \]

    \[  A^{- 1}\mathbf{=}\frac{\begin{bmatrix}\mathbf{- 6} & \mathbf{22} & \mathbf{31} \\\mathbf{37} & \mathbf{- 19} & \mathbf{13} \\\mathbf{14} & \mathbf{7} & \mathbf{- 14} \\\end{bmatrix}}{\mathbf{175}}\ \]

    \[ A^{- 1} = \begin{bmatrix}\mathbf{-}\frac{\mathbf{6}}{\mathbf{175}} & \frac{\mathbf{22}}{\mathbf{175}} & \frac{\mathbf{31}}{\mathbf{175}} \\\frac{\mathbf{37}}{\mathbf{175}} & \mathbf{-}\frac{\mathbf{19}}{\mathbf{175}} & \frac{\mathbf{13}}{\mathbf{175}} \\\frac{\mathbf{14}}{\mathbf{175}} & \frac{\mathbf{7}}{\mathbf{175}} & \mathbf{-}\frac{\mathbf{14}}{\mathbf{175}} \\\end{bmatrix}\ \]

Q.6 (a) Solve for x and y: 4x – 3y = 10, 5x – 7y = 6

Solution:

4x – 3y = 10 (i)

5x – 7y = 6 (ii)

Multiply equation (i) by 5 and (ii) by 4, subtract & get:

    \[  20x\ -\ 15y\ = \ 50\ \]

    \[ \pm 20x\ \mp \ 28y\ = \ \pm 24\  \]

    \[ 13y = 26\  \]

    \[ y = \frac{26}{13} = 2\  \]

Put y =2 in equation (i) to get the value of x

    \[  4x\ -\ 3y\ = \ 10\ (i)\ \]

    \[ 4x\ -\ 3(2)\ = \ 10\  \]

    \[ 4x\ -\ 6\ = 10\  \]

    \[ 4x\ = \ 10\ + \ 6\  \]

    \[ 4x\ = \ 16\  \]

    \[  x = \frac{16}{4} = 4\ \]

    \[  x = 4,\ y = 2\ \]

Prove:

4x – 3y = 10 (i)

4(4) – 3(2) = 10

10 = 10

(b) The area of a rectangular plot of land fenced all round is 2000 sq. yards and the total length of fencing is 180 sq. yards. Find the length and width of plot.

Solution:

Let the length of the plot be x

Let the width of the plot be y

Area = Length x Width

Area = 2000 sq. yards

Length of fencing = 180 sq. yards = Perimeter

Equation will be:

    \[ Area\  = \ xy\ \]

    \[ xy = \ 2000(i)\ \]

    \[ 2x\  + \ 2y\  = \ 180\ \]

    \[ 2(x + y)\  = \ 180\ \]

    \[ x + y = \frac{180}{2}\ \]

    \[ x + y = 90\ \ (ii)\ \]

    \[ y = 90 - x\ \ \]

Put y = 90 – x into equation (i)

    \[ xy = 2000\ \ (i)\ \]

    \[ x(90 - x) = 2000\ \]

    \[ 90x - x^{2} = 2000\ \]

    \[ 90x - x^{2} - 2000 = 0\ \]

    \[ - x^{2} + 90x - 2000 = 0\ or\ \]

    \[ x^{2} - 90x + 2000 = 0\ \]

    \[ a = 1,\ b = - 90,\ c = 2000\ \]

    \[ \frac{- b \pm \sqrt{b^{2} - 4ac}}{2a}\ \ \]

    \[ \frac{- ( - 90) \pm \sqrt{{( - 90)}^{2} - 4(1)(2000)}}{2(1)}\ \ \]

    \[ \frac{90 \pm \sqrt{8100 - 8000}}{2}\ \ \]

    \[ \frac{90 \pm 10}{2}\ \ \]

    \[ \frac{90 + 10}{2} = 50\ \&\ \frac{90 - 10}{2} = 40\ \]

    \[ Put\ x = 50\ \]

    \[ y = 90 - x\ \ \]

    \[ y = 90 - 50\ \ \]

    \[ y = 40\ \]

    \[ Put\ y\  = \ 40\ \]

    \[ y = 90 - x\ \ \]

    \[ 40 = 90 - x\ \ \]

    \[ - x = 40 - 90\ \]

    \[ - x = - 50\ \]

    \[ x = 50\ \]

Hence

Length of the plot be x = 50

Width of the plot be y = 40

Q.7 (a) Show that the sum of geometric series of ten terms…..

Q.7 (a) Show that the sum of geometric series of ten terms:

    \[  1.\ - \frac{1}{2},\frac{1}{4},\ - \frac{1}{8},,\frac{1}{16},\ - \frac{1}{32},\ldots\ldots is\ \frac{341}{512}\ \]

Solution:

    \[ a = 1,\ n = 10\  \]

    \[  Common\ Ratio\ r = \ \frac{- \frac{1}{2}}{1} = - \frac{1}{2}\ \]

    \[  Since\ |r| = \left| - \frac{1}{2} \right| < 1\ \]

First we have to find 10th term as:

    \[  an = a1r^{n - 1}\ \]

    \[  a10 = 1\left( - \frac{1}{2} \right)^{10 - 1}\ \]

    \[  a10 = \left( - \frac{1}{2} \right)^{9} = - \frac{1}{512}\ \]

Applicable Formula

    \[ S_{n} = \frac{a1 - a1r^{n}}{1 - r}\ where\ r < 1\ \]

    \[ S_{10} = \frac{1 - 1{( - \frac{1}{2})}^{10}}{1 - ( - \frac{1}{2})}\ \]

    \[ S_{10} = \frac{1 - \frac{1}{1024}}{1 + \frac{1}{2}}\ \]

    \[ S_{10} = \frac{\frac{1023}{1024}}{\frac{3}{2}}\ \]

    \[ S_{10} = \frac{1023}{1024} \times \frac{2}{3}\ \]

    \[ S_{10} = \frac{2046}{3072}\ \]

    \[ S_{10} = \frac{341}{512}\ \]

(b) A company offers two alternatives for the payment of salary for the post of a high executive. Either one may receive Rs. 240,000 per year or Rs. 100 in the first month, Rs. 200 in the second month, Rs. 400 in the third month and so on. Which of the two alternatives should be prefer…..

Solution:

First Alternative = 240,000 per year

Second alternative = 100, 200, 400…..12 terms

    \[ a1 = 100,\ n = 12\ \]

    \[ Common\ Ratio\ r = \ \frac{200}{100} = 2\ \]

    \[ Since\ |r| = |2| > 1\ \]

Applicable Formula

    \[ S_{n} = \frac{ra_{n} - a1}{r - 1}\ where\ r > 1\ \]

First we have to find 12th term:

    \[ a_{n} = a1r^{n - 1}\ \]

    \[ a_{12} = 100 \times 2^{12 - 1}\ \]

    \[ a_{12} = 100 \times 2^{11}\ \]

    \[ a_{12} = 204800\ \]

Applicable Formula

    \[ S_{n} = \frac{ra_{n} - a1}{r - 1}\ where\ r > 1\ \]

    \[ S_{12} = \frac{ra_{12} - a1}{r - 1}\ \ \]

    \[ S_{12} = \frac{2(204800) - 100}{2 - 1}\ \ \]

    \[ S_{12} = 409500\ \]

Since the second option is greater than 1

409500>240,000 So Executive should prefer 2nd option

Q.8 (a) Find the compound interest on Rs. 4500 in 3 years. If the rate of interest is 4% for the first year, 5% for the 2nd year and 6% for the 3rd year.

Solution:

    \[ \emph{Principal Amount (P) = 4500} \]

    \[ \emph{Interest Rate (i) for 1\textsuperscript{st} year = 4\%} \]

    \[ \emph{Interest Rate (i) for 2\textsuperscript{nd} year = 5\%} \]

    \[ \emph{Interest Rate (i) for 3\textsuperscript{rd} year = 6\%} \]

Calculation

    \[ \mathbf{Amount = P(1 +}\mathbf{i)}^{\mathbf{n}}\ \]

    \[ Amount\ 1st\ year = 4500(1 + {0.04)}^{1} = 4680\ \]

    \[ Amount\ 2nd\ year = 4680(1 + {0.05)}^{1} = 4914\ \]

    \[ Amount\ 3rd\ year = 4914(1 + {0.06)}^{1} = 5208.84\ \]

Compound Interest = 5208.84 – 4500 = 708.84

(b)  Find the accumulated value of Rs. 5000 invested at the end of each quarter for 5 years at 8% compounded quarterly.

Solution:

    \[  F.V\ = \ ?,\ R\ = \ 5000,\ n = 5 \times 4 = 20,i = \frac{8}{4} = 2\% = 0.02\ \]

Case = Ordinary Annuity, Quarterly Case

    \[  F.V = R\left\lbrack \frac{(1 + i)^{n} - 1}{i} \right\rbrack\ \]

    \[ F.V = 5000\left\lbrack \frac{(1 + 0.02)^{20} - 1}{0.02} \right\rbrack\  \]

    \[  F.V = 5000(24.2973)\ \]

    \[  F.V = 121487\ \]

Business Statistics and Mathematics Solved Paper 2012, Punjab University, BCOM,ADC I

Business Statistics & Mathematics, Solved Paper 2011, Punjab University, BCOM,ADCI

Business Statistics & Mathematics, Solved Paper 2010, Punjab University, BCOM, ADCI

Business Statistics & Mathematics, Solved Paper 2009, Punjab University, BCOM, ADCI

Business Mathematics and Statistics, Solved Paper 2008, Punjab University, BCOM,ADCI

Business Mathematics and Statistics, Solved Paper 2007, Punjab University, BCOM,ADCI

Introduction to Statistics Basic Important Concepts

Measures of Central Tendency, Arithmetic Mean, Median, Mode, Harmonic, Geometric Mean

Correlation Coefficient, Properties, Types, Important Formulas for Correlation Coefficient

Leave a Comment

Your email address will not be published. Required fields are marked *